Chapter 6 Flashcards
A 55-year-old man with seated office blood pressure (BP) readings of 138/89 mm Hg and
136/84 mm Hg is asked to return in 2 weeks for repeat measurements, which are 138/88 mm Hg
and 134/82 mm Hg. Which of the following classifies his BP per the 2017 ACC/AHA
guidelines?
A. Stage 2 hypertension
B. Stage 1 hypertension
C. Elevated BP
D. Optimal BP
Answer: B
Correct. The patient’s average BP falls within the range of systolic BP of 130 to 139
mm Hg or diastolic BP of 80 to 89 mm Hg, defined as stage 1 hypertension.
A 45-year-old woman who has resistant hypertension, heart failure with reduced ejection
fraction, type 2 diabetes, and dyslipidemia comes to the clinic for a medication management visit. Her current medications include enalapril/hydrochlorothiazide (Vaseretic), metoprolol succinate, hydralazine, isosorbide dinitrate, metformin, and atorvastatin. She has been adherent to all her medications. She complains of recently developed fatigue, arthralgia, and mild peripheral edema, and a butterfly rash across her nose and upper cheeks. Which of the following
statements is correct?
A. The most likely medication causing this adverse reaction is the angiotensin-converting
enzyme inhibitor, enalapril.
B. This is likely to be a dose-independent adverse drug reaction.
C. Testing for N-acetyltransferase-1 (NAT1) and NAT2 genes may help to prevent this adverse
effect.
D. Discontinue all medications due to unknown etiology of this reaction.
Option C: Correct. FDA listed NAT1 and NAT2 genes as biomarkers for the effectiveness and/or
toxicity of hydralazine.
A 55-year-old woman has a history of left ventricular hypertrophy with a left ventricular
ejection fraction of 55% (0.55). She has had hypertension for 10 years and is currently taking
chlorthalidone 25 mg daily, metoprolol succinate 50 mg daily, and amlodipine 2.5 mg daily. Her
averaged BP is 152/94 mm Hg with a heart rate of 54 beats/min. Her physical examination is
unremarkable, and basic metabolic panel reveals serum creatinine of 0.8 mg/dL (71 µmol/L) and
potassium of 3.9 mEq/L (mmol/L). She reports allergies to fosinopril and aspirin. Which of the
following represents the optimal course of action?
A. Increase amlodipine to 5 mg and have her take it at bedtime
B. Increase metoprolol succinate to 100 mg daily
C. Add lisinopril 5 mg daily
D. Add spironolactone 50 mg daily
Option A: Correct. Amlodipine may be further titrated to a maximum dose of 10 mg daily to
achieve BP control rather than adding new medications.
A 58-year-old man presents to urgent care with chief complaints of severe headache,
confusion, and blurred vision. Upon examination, his vital signs and relevant laboratory values include BP 218/124 mm Hg; HR 92 beats/min; RR 18 breaths/min; serum potassium 4.8 mEq/L (mmol/L); serum creatinine 2.4 mg/dL (212 µmol/L); blood urea nitrogen 45 mg/dL (16.1mmol/L); serum glucose 145 mg/dL (8.0 mmol/L); hemoglobin A 1C 6.4% (0.064; 46 mmol/mol Hb); AST 28 U/L (0.47 µkat/L); ALT 20 U/L (0.33 µkat/L); urinalysis shows positive of
proteins and negative of ketones. He has medical history of hypertension but no history of renalor hepatic impairment. According to the pharmacist, his antihypertensive medications are overdue for a refill by 2 months. Which of the following statements is most accurate?
A. He is experiencing hypertensive urgency.
B. Goal is to reduce SBP by up to 25% within an hour.
C. Goal is to reduce SBP by 25% to 50% within an hour.
D. Administration of short-acting oral antihypertensive such as labetalol or clonidine is
appropriate.
Option B: Correct.
A 57-year-old woman has type 2 diabetes, morbid obesity, and hypertension. She is currently taking only lisinopril 20 mg daily, and her office BPs are consistently at goal less than 130/80mm Hg, but her home readings are significantly higher. Which of the following is a possible explanation for her elevated home readings?
A. Her home BP cuff is too small.
B. She has white coat hypertension.
C. Her home BP cuff is too large.
D. She checks her BP immediately after exercise.
Option A: Correct. It is important to have the correct BP cuff size as cuffs that are too small will
result in falsely elevated BP readings.
A 72-year-old black man with history of heart failure (NYHA class III symptoms) with
reduced left ventricular ejection fraction, dyslipidemia, and peripheral arterial disease presents for risk factor reduction follow-up. His weight does not change from the baseline and he lacks any signs and symptoms for fluid retention. His BP in clinic is 142/75 mm Hg initially and 140/77 mm Hg on repeat. Current medications include bisoprolol 10 mg once daily and valsartan 160 mg twice daily along with furosemide 20 mg as needed for edema. Current laboratory values are stable and within normal limits with serum creatinine of 1.4 mg/dL (124 µmol/L) and potassium of 5.1 mEq/L (mmol/L). Which of the following additions to his medication regimen would be most appropriate at this time?
A. Amlodipine
B. Furosemide on a regular daily basis
C. Hydralazine/isosorbide dinitrate
D. Minoxidil
Option C: Correct. The combination of nitrates and hydralazine is the best option due to a
mortality benefit in black patients with symptomatic HFrEF.
A 32-year-old woman is 20 weeks pregnant and has a history of gestational diabetes. She
presents with an average BP of 154/96 mm Hg and a heart rate of 60 beats/min. Her laboratory
results are remarkable for proteinuria, elevated serum uric acid, and low potassium. Which of the following presents the most appropriate course of action?
A. Closely monitor her BP and provide supportive care
B. Start losartan 50 mg daily while monitoring BP
C. Start methyldopa 250 mg every 6 hours while monitoring BP
D. Start labetalol 100 mg every 12 hours while monitoring BP
Option C: Correct. Methyldopa is a first-line agent in pregnancy and is the best answer choice.
An 80-year-old man presents to your clinic today with average BP of 142/84 mm Hg. The
patient is a community-dwelling adult with no significant past medical history and is in
agreement to starting an antihypertensive agent today. Which of the following is the preferred
guideline-based BP goal for this patient?
A. < 130/80 mm Hg
B. <135/80 mm Hg
C. < 140/90 mm Hg
D. < 150/90 mm Hg
Option A: Correct. Based on 2017 AHA/ACC guideline recommendations given ambulatory
community-dwelling adult.
A 79-year-old man with a past medical history significant for hypertension and dyslipidemia
returns for a 3-week follow-up on BP readings and laboratory values after initiation of lisinopril
10 mg daily for uncontrolled hypertension. Today, average BP has improved from 151/93 mm Hg to 128/71 mm Hg with an increase in serum creatinine from 1.2 mg/dL (106 μmol/L) to 1.4 mg/dL (124 μmol/L). Which of the following interventions would be recommended at this time?
A. Continue lisinopril 10 mg once daily
B. Stop lisinopril and initiate amlodipine 5 mg daily
C. Decrease lisinopril to 5 mg daily
D. Stop lisinopril and initiate losartan 25 mg daily
Option A: Correct. Given patient’s BP is controlled and approximate 17% rise in serum
creatinine is less than 30%, thus acceptable post-ACE-I or ARB initiation; no indication for
further changes.
- A 57-year-old man with a past medical history of hypertension, type 2 diabetes, and
dyslipidemia returns to clinic for follow-up on BP readings after recent initiation of amlodipine.
His current medications include amlodipine 5 mg daily, metformin, and atorvastatin. Per his
office BP measurements and reported HBPM, there has been minimal change in his BP and it
remains uncontrolled. Patient also reports new-onset edema of the lower extremities. Which of
the following would be the most appropriate intervention at this time?
A. Continue amlodipine and initiate chlorthalidone 12.5 mg daily
B. Continue amlodipine and initiate furosemide 20 mg daily
C. Stop amlodipine and initiate lisinopril 10 mg daily
D. Stop amlodipine and initiate atenolol 25 mg daily
Option C: Correct. Based on options available, patient likely with adverse reaction from
amlodipine and has had minimal benefit on BP lowering, thus change to alternative first-line
agent is indicated.
A 55-year-old woman with a past medical history significant for hypertension and 1+
bilateral edema is admitted due to decompensated heart failure with reduced ejection fraction of 30% to 35% (0.3–0.35). Her current medications include carvedilol 25 mg twice daily, furosemide 20 mg once daily, and amlodipine 5 mg daily. Her BP is currently controlled on this regimen with an average BP of 118/77 mm Hg and heart rate of 68 beats/min. She denies any allergies. What intervention would be recommended at this time?
A. No interventions necessary
B. Initiate lisinopril 10 mg daily
C. Discontinue amlodipine and initiate enalapril 5 mg once daily
D. Discontinue amlodipine and initiate chlorthalidone 12.5 mg daily
Option C: Correct. Because of the patient’s history of heart failure with reduced ejection fraction, she has a compelling indication of ACE-I therapy, and given BP control and edema at present, it would be most reasonable to stop amlodipine and start enalapril.
A 67-year-old woman with a past medical history significant for hypertension, dyslipidemia,
and well-controlled asthma presents to clinic for postdischarge follow-up status post-MI. Her
vital signs include an average BP of 139/87 mm Hg and HR of 85 beats/min. Her current
medication list includes lisinopril, rosuvastatin, aspirin, clopidogrel, and budesonide-formoterol.
Which of the following β-blockers would be recommended to initiate today?
A. Propranolol
B. Labetalol
C. Pindolol
D. Bisoprolol
Option D: Correct. Bisoprolol is considered the most β 1 -selective β-blocker and is most
reasonable in a patient post-MI with controlled asthma.
What daytime ABPM target would be most reasonable to apply when assessing BP control
with ABPM?
A. < 120/70 mm Hg
B. < 125/75 mm Hg
C. < 130/80 mm Hg
D. < 135/85 mm Hg
Option C: Correct. Based on corresponding value estimates of clinic and daytime ABPM values,
a clinic target of less than 130/80 mm Hg would be expected to correlate with daytime ABPM
value of 130/80 mm Hg.
- Initiation of hydrochlorothiazide can result in which of the following electrolyte disturbance?
A. Hyperkalemia
B. Hypouricemia
C. Hypercalcemia
D. Hypernatremia
Option C: Correct. Can occur with thiazide therapy.
A 56-year-old woman presents to your clinic today with an average BP of 137/88 mm Hg.
She has no significant past medical history and is currently taking a multivitamin daily.
Significant vital signs and laboratory values include HR 79 beats/min, BMI 30.1 kg/m 2 ,
potassium 4.1 mEq/L (mmol/L), and an estimated GFR of 78 mL/min/1.73 m 2 . She has an estimated 10-year ASCVD risk of 7.5%. Which of the following would be the most appropriate
intervention at this time?
A. Recommend weight loss, exercise, and a heart-healthy diet
B. Initiate lisinopril 10 mg daily
C. Initiate amlodipine 5 mg daily
D. Initiate labetalol 100 mg twice daily
Option A: Correct. BP is above target but given ASCVD risk is less than 10%, patient does not
meet threshold for initiation of antihypertensive therapy at this point, and nonpharmacologic
interventions are most appropriate for this patient.